You are investing $5,000 and can invest for 2 years or 3 years at 1.75% and 1.25% interest rates, respectively. Which earns more interest

Answers

Answer 1

Answer:

The 3 years investment earns more interest

Step-by-step explanation:

Given

Principal, P = $5,000

Required

Determine which earns more interest

When Rate = 1.75% and Year = 2

Interest is as follows;

[tex]I = \frac{PRT}{100}[/tex]

Substitute 1.75 for R, 5000 for P and 2 for T

[tex]I = \frac{5000 * 1.75 * 2}{100}[/tex]

[tex]I = \frac{17500}{100}[/tex]

[tex]I = \$175[/tex]

When Rate = 1.25% and Year = 3

Interest is as follows;

[tex]I = \frac{PRT}{100}[/tex]

Substitute 1.25 for R, 5000 for P and 3 for T

[tex]I = \frac{5000 * 1.25 * 3}{100}[/tex]

[tex]I = \frac{18750}{100}[/tex]

[tex]I = \$187.5[/tex]

Comparing the interest of both investments, the 3 years investment earns more interest


Related Questions

Finding the perimeter or area of a rectangle given one of th
The length of a rectangle six times its width.
If the area of the rectangle is 150 cm”, find its perimeter.

Answers

Answer:

The answer is 70cm

Step-by-step explanation:

Perimeter of a rectangle = 2l + 2w

Area of a rectangle = l × w

where

l is the length

w is the width

From the question

The length of a rectangle six times its width which is written as

l = 6w

Area = 150cm²

Substitute these values into the formula for finding the area

That's

150 = 6w²

Divide both sides by 6

w² = 25

Find the square root of both sides

width = 5cm

Substitute this value into l = 6w

That's

l = 6(5)

length = 30cm

So the perimeter of the rectangle is

2(30) + 2(5)

= 60 + 10

= 70cm

Hope this helps you

9x - 3 -8x = 7 - x what is x Please answer ASAP, is urgent!!

Answers

Solve

Let's solve your equation step-by-step.

9x−3−8x=7−x

Step 1: Simplify both sides of the equation.

9x−3−8x=7−x

9x+−3+−8x=7+−x

(9x+−8x)+(−3)=−x+7(Combine Like Terms)

x+−3=−x+7

x−3=−x+7

Step 2: Add x to both sides.

x−3+x=−x+7+x

2x−3=7

Step 3: Add 3 to both sides.

2x−3+3=7+3

2x=10

Step 4: Divide both sides by 2.

2x

2

=

10

2

x=5

Answer:

x=5

Answer:

Hope this is easier, good luck.

2000 people attended a baseball game. 1300 of the people attending supported the home team, while 700 supported the visiting team. What percentage of people attending supported the home team?

Answers

Answer:

Percentage of home team supporters =65%

Percentage of visiting team supporters =35%

Step-by-step explanation:

Total attendees=2,000 people

Home team supporters=1,300

Visiting team supporters=700

What percentage of people attending supported the home team?

Percentage of people attending who supported the home team = home team supporters / total attendees × 100

=1,300/2,000 × 100

=0.65 × 100

=65%

Visiting team supporters = visiting team supporters / total attendees

× 100

=700/2000 × 100

=0.35 × 100

=35%

Alternatively,

Visiting team supporters = percentage of total attendees - percentage of home team supporters

=100% - 65%

=35%

Solve the equation using square roots x^2+20=4

Answers

Answer:

Step-by-step explanation:

x^2+20=4 first isolate the variable by subtracting 20 on both sides.

x^2=-16 again isolate the variable but this time you square root both sides.

[tex]\sqrt{x}^2[/tex]=[tex]\sqrt{-16[/tex] then simplify

x= ±4

What is the area, in square meters, of the shaded part of the rectangle shown below?

Answers

Answer:

C) 100 cm²

Step-by-step explanation:

(14*6)/2*10

20/2*10

10*10

100

The area of the given shaded part of the rectangle is 100 square meters as shown.

What is the area of a triangle?

The entire space filled by a triangle's three sides in a two-dimensional plane is defined as its area.

The fundamental formula for calculating the area of a triangle is A = 1/2 b h.

The area of the shaded part = area of the rectangle -  area of the triangle

The area of the shaded part = 14 × 10 - (1/2) × 8 × 10

The area of the shaded part = 140 - 80/2

The area of the shaded part = 140 - 40

Apply the subtraction operation, and we get

The area of the shaded part = 100 meters²

Thus, the area of the given shaded part of the rectangle is 100 square meters.

Learn more about the triangles here:

https://brainly.com/question/17997149

#SPJ3

This test statistic leads to a decision to...

reject the null

accept the null

fail to reject the null



As such, the final conclusion is that...

There is sufficient evidence to warrant rejection of the claim that the population mean is not equal to 88.9.

There is not sufficient evidence to warrant rejection of the claim that the population mean is not equal to 88.9.

The sample data support the claim that the population mean is not equal to 88.9.

There is not sufficient sample evidence to support the claim that the population mean is not equal to 88.9.

Answers

Answer:

There is not sufficient sample evidence to support the claim that the population mean is not equal to 88.9.

Step-by-step explanation:

We are given the following hypothesis below;

Let [tex]\mu[/tex] = population mean.

So, Null Hypothesis, [tex]H_0[/tex] : [tex]\mu[/tex] = 88.9      {means that the population mean is equal to 88.9}

Alternate Hypothesis, [tex]H_A[/tex] : [tex]\mu\neq[/tex] 88.9     {means that the population mean is different from 88.9}

The test statistics that will be used here is One-sample t-test statistics because we don't know about population standard deviation;

                             T.S.  =  [tex]\frac{\bar X-\mu}{\frac{s}{\sqrt{n} } }[/tex]  ~  [tex]t_n_-_1[/tex]

where, [tex]\bar X[/tex] = sample mean = 81.3

             s = sample standard deviation = 13.4

            n = sample size = 7

So, the test statistics =  [tex]\frac{81.3-88.9}{\frac{13.4}{\sqrt{7} } }[/tex]   ~ [tex]t_6[/tex]

                                     =  -1.501

The value of t-test statistics is -1.501.

Also, the P-value of the test statistics is given by;

                    P-value = P([tex]t_6[/tex] < -1.501) = 0.094

Since the P-value of our test statistics is more than the level of significance as 0.094 > 0.01, so we have insufficient evidence to reject our null hypothesis as the test statistics will not fall in the rejection region.

Therefore, we conclude that the population mean is equal to 88.9.

How to convert 2cm to feet?

Answers

Answer:

Divide by 30.48: It would be 0.0656168 feet.

Step-by-step explanation:

Answer:

0.0656

Step-by-step explanation:

2.54 cm = 1 in

12 in = 1 ft

2.54 * 12 = 30.48

2/30.48 = 0.0656167979

BRAINLIEST
Given that 104 = 10,000, write this in logarithm form.

Answers

Answer:

[tex]log_{10}[/tex] 10000 = 4

Step-by-step explanation:

Using the rule of logarithms

[tex]log_{b}[/tex] x = n ⇔ x = [tex]b^{n}[/tex]

Here b = 10, n = 4 and x = 10000, thus

[tex]log_{10}[/tex] 10000 = 4 ← in logarithmic form

that is [tex]10^{4}[/tex] = 10000 ← in exponential form

Please Help! The point (8, -2) satisfies the equation of which line? (1) y+2=2(x+8) (2) y-2=2(x-8) (3) y+2=2(x-8) (4) y-2=2(x+8)

Answers

Answer:

(3) y+2=2(x-8)

Step-by-step explanation:

Substitute the point into the equation and see if it is true

(8,-2)

(1) y+2=2(x+8)

    -2+2 = 2(8+8)

    0 = 2(16)

False

(2) y-2=2(x-8)

  -2-2 = 2(8-8)

  -4 =2 (0)

  False

(3) y+2=2(x-8)

  -2+2 = 2( 8-8)

  0 = 2(0)

 True

(4) y-2=2(x+8)

  -2-2 = 2(8+8)

  -4 = 2(16)

False

Answer:

[tex]\boxed{y+2=2(x-8) }[/tex]

Step-by-step explanation:

[tex]x=8[/tex]

[tex]y=-2[/tex]

[tex]\sf Check \ the \ third \ option.[/tex]

[tex]-2+2=2(8-8)[/tex]

[tex]\sf Both \ sides \ must \ be \ equal.[/tex]

[tex]0=2(0)[/tex]

[tex]0=0[/tex]

Within what values will 95.44 percent of sample means of this process fall, if samples of n = 8 are taken and the process is in control (random)?

Answers

This question is incomplete, here is the complete question:

Specifications for a part for a DVD player state that the part should weigh between 25.2 and 26.2 ounces. The process that produces the parts has a mean of 25.7 ounces and a standard deviation of .25 ounce. The distribution of output is normal. Use Table-A

a) What percentage of parts will not meet the weight specs? (Round your "z" value and final answer to 2 decimal places.

b) Within what values will 95.44 percent of sample means of this process fall, if samples of n = 8 are taken and the process is in control (random)

Answer:

a) What percentage of parts will not meet the weight specs = 4.56%

b) values within which 95.44 percent of sample means of this process falls are;

UCL = 25.88 ounces

LCL = 25.52 ounces

Step-by-step explanation:

Given that;

Mean u = 25.7 ounces

Std deviation = 0.25 ounces

a)

Z-score (Upper) = (X- u) / s = (26.2 - 25.7) / 0.25 = 2

Z-score (Lower) = ( 25.2-25.7 ) / 0.25 = -2

using the T - table

For Z = 2.0

the area in the tail of the curve to the right of the mean (upper) = 0.4772

therefore;

Number of defective = 0.5000 - 0.4772 = 0.0228

These are errors on one side of normal distribution.

To get the total error, we say

Total error = 2 × 0.0228

Total error = 0.0456 ≈ 4.56% ( 2 decimal place )

b)

given that;

n = 8,

standard deviation = 0.25 ounce

Standard deviation of X = Std deviation / √n

= 0.25 /√8 = 0.088

Now for 95.44% of confidence interval, Z = 2

UCL = Mean + Z × Standard deviation of X

= 25.7 + 2 × 0.088

= 25.88 ounces

LCL = Mean - Z × Standard deviation of X

= 25.7 - 2 × 0.088

= 25.52 ounces

Stefan rode 32.95 miles.
Ben rode 25 4/5 miles. How many more miles did stefan ride than ben?​

Answers

Answer:

7.15 miles

Step-by-step explanation:

4/5 of a mile is equivalent to .8 miles.

32.95

-25.8

7.15

Answer:

Step-by-step explanation:

39.95 - 25.80 = 7.15 miles

You drive 15 miles in 0.1hours . How fast did you travel if 8=d/t

Answers

Answer:

150

Step-by-step explanation:

[tex]distance = 15 miles\\time = 0.1 hours\\\\Speed = \frac{Distance}{time}\\ Speed = \frac{15}{0.1}\\ Speed =150[/tex]

Answer:

[tex]150mph[/tex]

Step-by-step explanation:

Given:

s=15miles

t=0.1hours

Required:

v=?

Formula:

[tex]v = \frac{s}{t} [/tex]

Solution:

[tex]v = \frac{s}{t} = \frac{15m}{0.1h} = \frac{150m}{1h} = 150mph[/tex]

Hope this helps ;) ❤❤❤

Help Me With This
show work​

Answers

Answer:

1. Make a list of activities and the number of students:

Watching TV: 32

Talking on the phone: 41

Video games: 24

Reading: 15

2. Then combine the data in a bar graph as shown in the picture


[tex]( \frac{3}{4} - \frac{2}{3} ) \times 1 \frac{1}{5} [/tex]

Answers

Answer: 0.1 or 1/10

Step-by-step explanation:

[tex]\left(\frac{3}{4}-\frac{2}{3}\right)\cdot \:1\frac{1}{5}[/tex]

[tex]1\frac{1}{5}=\frac{6}{5}[/tex]

[tex]\left(\frac{3}{4}-\frac{2}{3}\right)\cdot \frac{6}{5}[/tex]

  [tex]\frac{3}{4}-\frac{2}{3}[/tex]    [tex]=\frac{9}{12}-\frac{8}{12}[/tex]

[tex]=\frac{1}{12}[/tex]

[tex]\frac{6}{5}\cdot \frac{1}{12}[/tex]

Cross, cancel common factor

[tex]\frac{1}{2}\cdot \frac{1}{5}[/tex]

[tex]=\frac{1}{10}[/tex]

Find y using the Angle Sum Theorem

Answers

Step-by-step explanation:

Hey, there!!

Look this figure, simply we find that;

In triangle ABC,

angle CBD is an exterior angle of a triangle.

and its measure is 90°

Then,

angle CBD= y +48° {sum of interior opposite angle is equal to exterior angle or from theorem}.

or, 90°= y + 48°

Shifting, 48° in left side,

90°-48°= y

Therefore, the value of y is 42°.

Hope it helps...

Ajar contains 4 red marbles numbered 1 to 4 and 10 blue marbles numbered 1 to 10. A marble is

drawn at random from the jar. Find the probability of the given event.

(a) The marble is red

Your answer is:

(b) The marble is odd-numbered

Your answer is:

(C) The marble is red or odd-numbered

Your answer is:

(d) The marble is blue or even-numbered

Your answer is:

Question Help M Message instructor

Answers

Answer:

a)2/7

b)1/2

c)9/14

d)6/7

Step-by-step explanation:

The jar contains 4 red marbles, numbered 1 to 4 which means

Red marbles = (R1) , (R2) , (R3) , (R4)

It also contains 10 blue marbles numbered 1 to 10 which means

Blue marbles = (B1) , (B2) , (B3) , (B4) , (B5) , (B6) , (B7) , (B8) , (B9) , (B10) .

We can calculate total marbles = 4red +10 blues

=14marbled

Therefore, total marbles= 14

The marbles that has even number = (R2) , (R4) ,(B2) , (B4) , (B6) , (B8) , (B10) =7

Total number of Blue marbles = 10

Blue and even marbles = 5

(a) The marble is red

P(The marble is red)=total number of red marbles/Total number of marbles

=4/14

=2/7

(b) The marble is odd-numbered

Blue marbles with odd number= (B1) , (B3) , (B5) , (B7) , (B9) ,

Red marbles with odd number = (R1) , (R3)

Number of odd numbered =(5+2)=7

P(marble is odd-numbered )= Number of odd numbered/ Total number of marbles

P(marble is odd-numbered )=7/14

=1/2

(C) The marble is red or odd-numbered?

Total number of red marbles = 14

Number of red and odd marbles = 2

The marbles that has odd number = (R1) , (R3) ,(B1) , (B3) , (B5) , (B7) , (B9) =7

n(red or even )= n(red) + n(odd)- n(red and odd)

=4+7-2

=9

P(red or odd numbered)= (number of red or odd)/(total number of the marble)

= 9/14

(d) The marble is blue or even-numbered?

Number of Blue and even marbles = 5

Total number of Blue marbles = 10

Number of blue that are even= 5

The marbles that has even number = (R2) , (R4) ,(B2) , (B4) , (B6) , (B8) , (B10)

=7

n(Blue or even )= n(Blue) + n(even)- n(Blue and even)

= 10+7-5 =12

Now , the probability the marble is blue or even numbered can be calculated as

P(blue or even numbered)= (number of Blue or even)/(total number of the marble)

= 12/14

= 6/7

Help please!!! Thank you

Answers

Answer:

B: 54

Step-by-step explanation:

for the first digit: 1 or 3 (2 choices)

for the second digit: 0, 1, or 3 (3 choices)

for the third digit: 0, 1, or 3 (3 choices)

for the forth digit: 0, 1, or 3 (3 choices)

2×3×3×3=54

Answer:

B) 54

Step-by-step explanation:

There are 3 numbers, but in the fourth positon (tens of thousands) if i put the zero no give value, then, in this position only have 2 options:

2*3*3*3 = 54

Determine the present value P that must be invested to have the future value A at simple interest rate r after time t.
A = $8000.00, r = 10.5%, t = 9 months
$
(Round up to the nearest cent as needed.)

Answers

Answer:

$7,415.99

Step-by-step explanation:

Hello, please consider the following.

[tex]P\cdot (1+\dfrac{10.5\%\cdot 9}{12})=A = 8000 \\\\P = \dfrac{8000}{(1+\dfrac{31.5}{400})}=\dfrac{8000}{1.07875}\\\\=7415.990730...[/tex]

So it gives $7,415.99

Thank you.

If the bathtub holds a total of 46.2 gallons, how many minutes would it take to fill the entire tub? Write an equation in one variable to help you solve the problem. The variable represents the unknown time in minutes.

Answers

Answer:

46.2÷m=x

Step-by-step explanation:

u divide the amount of water by the time it takes to fill up(m). Witch will equal the amount per minute (x).

16.5/min

time = m

gallons / minutes = rate

46.2  = 16.5 (m)

46.2 / 16.5 = 16.5 (m) / 16.5

2.8 = minutes

Ben is tiling the floor in his bathroom the area he is tiling is 4m times 2m each tiles measures 400mm times 400mm he has 45 tiles is that enough

Answers

Answer:

No, it's not enough

Step-by-step explanation:

Given

Tilling Dimension = 4m by 2m

Tile Dimension = 400mm by 400mm

Required

Determine the 45 tiles is enough

First;

The area of the tiling has to be calculated

[tex]Area = Length * Breadth[/tex]

[tex]Area = 4m * 2m[/tex]

[tex]Area = 8m^2[/tex]

Next, determine the area of the tile

[tex]Area = Length * Breadth[/tex]

[tex]Area = 400mm * 400mm[/tex]

Convert measurements to metres

[tex]Area = 0.4m* 04m[/tex]

[tex]Area = 0.16\ m^2[/tex]

Next, multiply the above area result by the number of files

[tex]Total = 0.16m^2 * 45[/tex]

[tex]Total = 7.2m^2[/tex]

Compare 7.2 to 8

Hence, we conclude that the 45 tiles of 400mm by 400 mm dimension is not enough to floor his bathroom

Evaluate 3h(2) + 2k(3) =

Answers

Answer:

6h + 6k

Step-by-step explanation:

[tex]3h\left(2\right)+2k\left(3\right)\\\\\mathrm{Remove\:parentheses}:\quad \left(a\right)=a\\\\=3h\times \:2+2k\times \:3\\\\\mathrm{Multiply\:the\:numbers:}\:3\times \:2=6\\\\=6h+2\times \:3k\\\\\mathrm{Multiply\:the\:numbers:}\:2\times \:3=6\\\\=6h+6k[/tex]

Answer:

Answers for E-dge-nuityyy

Step-by-step explanation:

(h + k)(2) = 5

(h – k)(3) = 9

Evaluate 3h(2) + 2k(3) = 17

10) An amount of $1500.00 is invested for 3 years at rate of 2% for the first year and 5%, for
the 2nd year and 6% for the 3rd year.
a) Calculate the interest amount you will get if this is simple interest?
b) How much more or less you will get if this is compound interest?

Answers

Answer:

the interest is 195dollars

When csc(Theta)sin(Theta) is simplified, what is the result? StartFraction 1 Over cosecant squared EndFraction StartFraction 1 Over sine squared EndFraction 0 1

Answers

Step-by-step explanation:

csc θ sin θ

(1 / sin θ) sin θ

1

The simplified value of the given expression comes to be 1.

The given expression is:

[tex]cosec\theta.sin\theta[/tex]

What is the trigonometric ratio [tex]cosec\theta[/tex]?

The trigonometric ratio [tex]cosec\theta[/tex] is the ratio of the hypotenuse to the opposite side. It is the inverse of [tex]sin\theta[/tex].

[tex]cosec\theta=\frac{1}{sin\theta}[/tex]

We know that [tex]cosec\theta=\frac{1}{sin\theta}[/tex]

So [tex]cosec\theta.sin\theta[/tex]

[tex]=\frac{1}{sin\theta} .sin\theta[/tex]

=1

So, the simplified value is 1.

Hence, the simplified value of the given expression comes to be 1.

To get more about trigonometric ratios visit:

https://brainly.com/question/24349828

The perimeter of a rectangle is 80 cm. Find the lengths of the sides of the rectangle giving the maximum area.Enter the answers for the lengths of the sides in increasing order.

Answers

Answer:

The lengths of the sides are 20 cm and 20 cm

Step-by-step explanation:

Given

Perimeter, P = 80cm

Represent the length and width with L and W, respectively;

[tex]P= 2*(L + B)[/tex]

Substitute 80 for P

[tex]80 = 2 * (L + B)[/tex]

Divide through by 2

[tex]40 = L + B[/tex]

[tex]L + B = 40[/tex]

Make L the subject of formula

[tex]L = 40 - B[/tex]

Area of a rectangle is calculated as thus;

[tex]Area = L * B[/tex]

Substitute 40 - B for L

[tex]Area = (40 - B) * B[/tex]

Express this as a function

[tex]A(B) = (40 - B)* B[/tex]

[tex](40 - B)* B = A(B)[/tex]

Set A(B) = 0 to determine the roots

Hence;

[tex](40 - B)* B = 0[/tex]

[tex]40 - B = 0[/tex] or [tex]B = 0[/tex]

[tex]40 = B[/tex] or [tex]B = 0[/tex]

[tex]B = 40[/tex] or [tex]B = 0[/tex]

The maximum area of a rectangle occurs at half the sum of the roots;

So;

[tex]B= \frac{B_1 + B_2}{2}[/tex]

[tex]B= \frac{40+0}{2}[/tex]

[tex]B= \frac{40}{2}[/tex]

[tex]B = 20[/tex]

Recall that [tex]L = 40 - B[/tex]

[tex]L = 40 - 20[/tex]

[tex]L = 20[/tex]

Hence the lengths of the sides are 20 cm and 20 cm

Assume that females have pulse rates that are normally distributed with a mean of μ=73.0 beats per minute and a standard deviation of σ=12.5 beats per minute. Complete parts​ (a) through​ (c) below.a. If 1 adult female is randomly​ selected, find the probability that her pulse rate is less than 76 beats per minute.b. If 25 adult females are randomly​ selected, find the probability that they have pulse rates with a mean less than 76 beats per minute.c. Why can the normal distribution be used in part​ (b), even though the sample size does not exceed​ 30?A. Since the mean pulse rate exceeds​ 30, the distribution of sample means is a normal distribution for any sample size.B. Since the distribution is of​ individuals, not sample​ means, the distribution is a normal distribution for any sample size.C. Since the distribution is of sample​ means, not​ individuals, the distribution is a normal distribution for any sample size.D. Since the original population has a normal​ distribution, the distribution of sample means is a normal distribution for any sample size.

Answers

Answer:

a. the probability that her pulse rate is less than 76 beats per minute is 0.5948

b. If 25 adult females are randomly​ selected,  the probability that they have pulse rates with a mean less than 76 beats per minute is 0.8849

c.   D. Since the original population has a normal​ distribution, the distribution of sample means is a normal distribution for any sample size.

Step-by-step explanation:

Given that:

Mean μ =73.0

Standard deviation σ =12.5

a. If 1 adult female is randomly​ selected, find the probability that her pulse rate is less than 76 beats per minute.

Let X represent the random variable that is normally distributed with a mean of 73.0 beats per minute and a standard deviation of 12.5 beats per minute.

Then : X [tex]\sim[/tex] N ( μ = 73.0 , σ = 12.5)

The probability that her pulse rate is less than 76 beats per minute can be computed as:

[tex]P(X < 76) = P(\dfrac{X-\mu}{\sigma}< \dfrac{X-\mu}{\sigma})[/tex]

[tex]P(X < 76) = P(\dfrac{76-\mu}{\sigma}< \dfrac{76-73}{12.5})[/tex]

[tex]P(X < 76) = P(Z< \dfrac{3}{12.5})[/tex]

[tex]P(X < 76) = P(Z< 0.24)[/tex]

From the standard normal distribution tables,

[tex]P(X < 76) = 0.5948[/tex]

Therefore , the probability that her pulse rate is less than 76 beats per minute is 0.5948

b.  If 25 adult females are randomly​ selected, find the probability that they have pulse rates with a mean less than 76 beats per minute.

now; we have a sample size n = 25

The probability can now be calculated as follows:

[tex]P(\overline X < 76) = P(\dfrac{\overline X-\mu}{\dfrac{\sigma}{\sqrt{n}}}< \dfrac{ \overline X-\mu}{\dfrac{\sigma}{\sqrt{n}}})[/tex]

[tex]P( \overline X < 76) = P(\dfrac{76-\mu}{\dfrac{\sigma}{\sqrt{n}}}< \dfrac{76-73}{\dfrac{12.5}{\sqrt{25}}})[/tex]

[tex]P( \overline X < 76) = P(Z< \dfrac{3}{\dfrac{12.5}{5}})[/tex]

[tex]P( \overline X < 76) = P(Z< 1.2)[/tex]

From the standard normal distribution tables,

[tex]P(\overline X < 76) = 0.8849[/tex]

c. Why can the normal distribution be used in part​ (b), even though the sample size does not exceed​ 30?

In order to determine the probability in part (b);  the  normal distribution is perfect to be used here even when the sample size does not exceed 30.

Therefore option D is correct.

Since the original population has a normal distribution, the distribution of sample means is a normal distribution for any sample size.

While walking from the car into your dormitory you dropped your engagement ring somewhere in the snow. The path is 30 feet long. You are distraught because the density of its location seems to be constant along this 30-foot route. a) What is the probability that the ring is within 12 feet of your car

Answers

Answer:

0.4

Step-by-step explanation:

we are required to find the probability that the ring is within 12 meters from nthe car.

we start by defining a random variable x to be the distance from the car. the car is the starting point.

x follows a normal distribution (0,30)

[tex]f(x)=\frac{1}{30}[/tex]

[tex]0<x<30[/tex]

probabilty of x ≤ 12

= [tex]\int\limits^a_ b{\frac{1}{30} } \, dx[/tex]

a = 12

b = 0

[tex]\frac{1}{30} *(12-0)[/tex]

[tex]\frac{12}{30} = 0.4[/tex]

therefore 0.4 is the probability that the ring is within 12 feet of your car.

In how many ways can a subcommittee of 6 students be chosen from a committee which consists of 10 senior members and 12 junior members if the team must consist of 4 senior members and 2 junior members?

Answers

Answer:

The number of ways is 13860 ways

Step-by-step explanation:

Given

Senior Members = 10

Junior Members = 12

Required

Number of ways of selecting 6 students students

The question lay emphasis on the keyword selection; this implies combination

From the question, we understand that

4 students are to be selected from senior members while 2 from junior members;

The number of ways is calculated as thus;

Ways = Ways of Selecting Senior Members * Ways of Selecting Junior Members

[tex]Ways = ^{10}C_4 * ^{12}C2[/tex]

[tex]Ways = \frac{10!}{(10-4)!4!)} * \frac{12!}{(12-2)!2!)}[/tex]

[tex]Ways = \frac{10!}{(6)!4!)} * \frac{12!}{(10)!2!)}[/tex]

[tex]Ways = \frac{10 * 9 * 8 * 7 *6!}{(6! * 4*3*2*1)} * \frac{12*11*10!}{(10!*2*1)}[/tex]

[tex]Ways = \frac{10 * 9 * 8 * 7}{4*3*2*1} * \frac{12*11}{2*1}[/tex]

[tex]Ways = \frac{5040}{24} * \frac{132}{2}[/tex]

[tex]Ways = 210 * 66[/tex]

[tex]Ways = 13860[/tex]

Hence, the number of ways is 13860 ways

Simplify: 9h-12h=54-23

A. 3h=-77

B.3h= 31

C.-3h= -31

D.-3h= 31

Answers

Answer:

c is the answer

Step-by-step explanation:

-3h = 31

-9h-12h = -3h

54-23= 31

Answer:

[tex]\boxed{C. -3h = 31}[/tex]

Step-by-step explanation:

Hey there!

9h - 12h = 54 - 23

Simplify

-3h = 31

C. -3h = 31

Hope this helps :)

5 STARS IF CORRECT! Can you translate a phrase or sentence into symbols? Explain the answer.

Answers

Answer:

See below.

Step-by-step explanation:

It depends on the sentence or phrase. If the sentence includes an operation of numbers or something related to comparing numbers, then maybe it can be translated into symbols. If the sentence or phrase has nothing to do with quantities, or operations or comparison of quantities, then probably it can't.

Examples:

1) The boy went for a walk.

There's nothing to translate into symbols in this case.

2) I had $10 in my bank account, then I deposited n dollars. Now I have $30 in my account.

In this case, I can translate the sentence into an equation.

10 + n = 30

For this problem, use the tables and charts shown in this section. (Use picture provided)
A United States Citizen returning to the States declares the following items at the customs office:
3 shirts at $8.50 each
2 dresses at $27.50 each
1 pair of gold cuff links at $17.50 per pair
If he has not used his duty free exemption yet, how much duty should he pay?
0 $0.00
$5.00
$10.00
$300

Answers

Answer:

0

Step-by-step explanation:

0 because there is a $100 duty free exemption.

answer:

For this problem, use the tables and charts shown in this section.  

A United States Citizen returning to the States declares the following items at the customs office:

3 shirts at $8.50 each

2 dresses at $27.50 each

1 pair of gold cuff links at $17.50 per pair

If he has not used his duty free exemption yet, how much duty should he pay?

$0.00 !

$5.00

$10.00

$300

Other Questions
Household members tend to have different preferences, but empirical evidence shows that overall, most households are Pareto efficient.a. Trueb. False The temperature dropped 15 degrees in an hour. If the starting temperature was 10 degrees, what was the final temperature? The value of y varies jointly with x and z. If y = 2 when z = 110 and x = 11, find the approximate value of y when x = 13 and z = 195. Mam, puedo salir a la calle? No, hija. ________. Est nublado y va a tronar. Hace mal tiempo Hace sol Hace buen tiempo Hace calor Which fractions are equal to 0.375? Cual de estas actividades no produce endorfinas ni opiaceos naturales? When you examine fluid that you have drawn from a blood vessel in someone's arm, you find that it is relatively high in carbon dioxide and waste products. Assuming that the fluid comes from a healthy individual, you conclude that the blood vessel from which you have drawn blood could have been ______. Kapil deposited Rs. 1600 in a bank on 1st January 2005. Find the amount in his bank account on 1st January 2006, if the bank pays interest at 8% per annum and the interest is calculated every year on 30th June and 31st December. If f(x)=x/2-3and g(x)=4x^2+x-4, find (f+g)(x) In capital rationing, alternative proposals that survive initial screening by cash payback and average rate of return methods are further analyzed using:________ On Wednesday at camp, Samuel went for a hike at 6:30 A.M. The hike took 2 hours and 15 minutes. As soon as he got back from the hike, Samuel played volleyball for 1 hour. What time did Samuel finish playing volleyball? Which type(s) of symmetry does the following object have?Select all that apply. According to John F. Galliher, legal definitions of criminality are arrived at through a __________ process. Inhibition of perceptual/attentional processes and inhibition of motor processes have both been proposed as possible mechanisms for which phenomenon? Give the values of a, b, and c needed to write the equation's general form.1/4x^2+5=0 which statement is true regarding the consequences of the Treaty of Guadalupe Hidalgo ? A) it ended the Mexican war of independenceB) it created the Transcontinental railroad through Texas C) it established Us control over in New Mexico Nevada and CaliforniaD) it solidified Mexican authority over Texas and the surrounding region Find the missing probability. P(A)=720,P(B)=35,P(AB)=21100 ,P(AB)=? Which expression represents the number 12.3 rewritten in a+bi form? 12.3+0i 1+12.3i 12.3+i 0+12.3i 56:31Rona mixes 2 pounds of meat with some chopped vegetables to make a mixture. She divides the mixture into 4 equal portions. Each portion weighs 3 pounds. Which equation and solution shows the total amount of chopped vegetables she used in the mixture? A company purchased a computer system at a cost of $34,000. The estimated useful life is 8 years, and the estimated residual value is $9,000. Assuming the company uses the double-declining-balance method, what is the depreciation expense for the second year